5.4 Teorema dei valori intermedi e funzioni continue su intervalli

annuncio pubblicitario
Università “Roma Tre” – L. Chierchia
5.4
13
Teorema dei valori intermedi e funzioni continue su intervalli
Studiamo, ora, il comportamento delle funzioni continue su intervalli15 . Innanzitutto, le
funzioni continue trasformano intervalli in intervalli:
Teorema 35 Se f ∈ C(I) con I intervallo, allora J = f (I) è un intervallo.
Dimostrazione Se J = {y} (ossia f è identicamente uguale a y), il teorema è vero (essendo
J = [y, y]). Siano, ora, y1 < y2 punti di J. Vogliamo far vedere che [y1 , y2 ] ⊆ J. Per definizione
di J, esistono x1 , x2 ∈ I tali che f (xi ) = yi (naturalmente, x1 �= x2 essendo y1 �= y2 ). Sia ora
y tale che y1 < y < y2 e si consideri la funzione continua F (x) := f (x) − y. Si osservi che
F (x1 ) = y1 − y < 0 < y2 − y = F (x2 ); quindi, per il teorema di esistenza di zeri per funzioni
continue applicato ad F su [a, b] ⊆ I con a := min{x1 , x2 } < b := max{x1 , x2 }, segue che
esiste x ∈ (a, b) tale che F (x) = 0 ossia f (x) = y.
Osservazione 36 (i) Essendo J = f (I) un intervallo, (inf J, sup J) ⊆ J (cf. nota 15) e
dunque:
Se inf J < y < sup J, allora esiste x ∈ I tale che y = f (x); in particolare, f assume ogni
valore tra f (x1 ) e f (x2 ), per ogni x1 , x2 ∈ I.
Questo risultato è noto come il “teorema dei valori intermedi”.
(ii) Dal Teorema 35 e dalla Proposizione 32 segue che se I = [a, b] è un intervallo compatto e
f ∈ C([a, b]) allora J = f ([a, b]) è compatto il che, per un intervallo, significa che J = [m, M ],
dunque:
f ∈ C([a, b]) =⇒ f ([a, b]) = [m, M ] ,
(18)
dove m e M sono il, rispettivamente, il minimo ed il massimo di f su [a, b].
15 Ricordiamo che, in generale, un intervallo I di R è un insieme tale che, se x < y sono punti di I, allora
[x, y] := {t ∈ R| x ≤ t ≤ y} ⊆ I. Gli intervalli di R sono, per definizione, gli insiemi “connessi” di R.
Esercizio 9∗ : si dimostri che I è un intervallo (o un insieme connesso) di R se e solo se non esistono due
insiemi non vuoti, aperti e disgiunti A e B tali che I ⊆ A ∪ B.
Esercizio 10: si dimostri che l’interno di un intervallo I è dato da (inf I, sup I).
14
Università “Roma Tre” – L. Chierchia
Discutiamo, ora, l’invertibilità di funzioni su intervalli.
Proposizione 37 Sia I un intervallo e f ∈ C(I). Allora, f è iniettiva se e solo se f è
strettamente monotona su I.
Dimostrazione Una funzione strettamente monotona è sempre iniettiva.
Assumiamo, ora, che f sia iniettiva e supponiamo, per assurdo, che non sia strettamente
monotona. Se f non è strettamente monotona, devono esistere tre punti x < y < z in I tali
che, se poniamo α := min{f (x), f (z)} e β := max{f (x), f (z)}, si ha o f (y) ≥ β oppure16
f (y) ≤ α. Ora, poiché stiamo assumendo l’iniettività di f si deve avere α < β e o f (y) > β
oppure f (y) < α. Supponiamo, ad esempio, che f (y) > β e sia ȳ tale che β < ȳ < f (y).
Poiché, f (x) ≤ β < ȳ < f (y), dal Teorema 35, segue che esiste x1 ∈ (x, y) tale che f (x1 ) = ȳ;
allo stesso modo, poiché f (z) ≤ β < ȳ < f (y), segue che esiste x2 ∈ (y, z) tale che f (x2 ) = ȳ
e quindi f (x1 ) = f (x2 ) con x1 < x2 , il che contraddice l’iniettività. Il caso f (y) < α si tratta
analogamente17 .
Proposizione 38 Sia I un intervallo e f ∈ C(I) iniettiva. Allora, la funzione inversa g =
f −1 è continua su18 J := f (I).
Dimostrazione Se I = {x}, la proposizione è banalmente vera. Assumiamo, ora, che inf I <
sup I. Per la Proposizione 37, f è strettamente monotona su I. Assumiamo che f sia strettamente crescente (altrimenti si consideri −f ). Consideriamo un punto ȳ interno a J. Allora,
esisteranno y1 , y2 ∈ J tali che y1 < ȳ < y2 . Siano xi = g(yi ) (i = 1, 2). Poiché f è strettamente crescente x1 < x2 e poiché f è continua, per la (18), f ([x1 , x2 ]) = [y1 , y2 ]. Quindi, per
la Proposizione 34, g è continua su [y1 , y2 ] ed in particolare è continua in ȳ.
Se ȳ è un estremo di J; allora o ȳ = max J oppure ȳ = min J. Nel primo caso possiamo
prendere y1 ∈ J con y1 < ȳ e, se f (x̄) = ȳ e f (x1 ) = y1 , sarà (per la stretta crescenza di f
e per (18)) f ([x1 , x̄]) = [y1 , ȳ] e (sempre per la Proposizione 34) g sarà continua in ȳ. Il caso
ȳ = min J è del tutto analogo.
16 Infatti: “f strettamente monotona su I” ⇐⇒ ∀ x < y < z punti di I si ha f (x) < f (y) < f (z) oppure
f (x) > f (y) > f (z) ⇐⇒ f (y) ∈ (α, β) dove α = min{f (x), f (z)} e β = max{f (x), f (z)}. Dunque “f non
strettamente monotona su I” ⇐⇒ ∃ < x < y < z in I tali che f (y) ∈
/ (α, β) ⇐⇒ f (y) ≥ β oppure f (y) ≤ α.
17 Esercizio 11.
18 J è un intervallo (Teorema 35).
Università “Roma Tre” – L. Chierchia
15
Osservazione 39 Si noti che l’ipotesi che I sia un intervallo è cruciale come mostra il
seguente esempio. Sia E = [0, 1) ∪ [2, 3] e sia f : E → R la seguente funzione continua:
�
x,
se 0 ≤ x < 1 ,
f (x) =
x − 1 , se 2 ≤ x ≤ 3 ,
che ha immagine I = f (E) = [1, 2]. Tale funzione è invertibile ma la sua inversa g : y ∈ I �→
g(y) = f −1 (y), che è data da
�
y,
se 0 ≤ x < 1 ,
g(y) =
y + 1 , se 1 ≤ y ≤ 2 ,
ha un salto in y = 1, essendo g(1−) = 1, g(1+) = 2. Si noti che il dominio di f , E, non è né
un intervallo, né un compatto.
Esempi e applicazioni:
E4 I risultati di questa sezione possono essere usati per dimostrare che19
cos x è strettamente decrescente in [0, π] e strettamente crescente in [π, 2π]; sen x è strettamente crescente in [−π/2, π/2] e strettamente decrescente in [π/23π/2].
E5 La funzione sen x con dominio [−π/2, π/2] è strettamente crescente; assume valore massimo uguale a 1 in x = π/2 e valore minimo uguale a −1 in x = −π/2, dunque, per la
(18), l’immagine di tale funzione è l’intervallo compatto [−1, 1]. La sua inversa prende il nome di Arcsen y (“arcoseno” o “ramo principale dell’arcoseno”): il suo dominio è [−1, 1] e la
sua immagine [−π/2, π/2] e per la Proposizione 34 è una funzione continua. Gli altri “rami”
dell’arcoseno si ottengono considerando altri domini (massimali) su cui il seno è invertibile:
ad esempio, [π/2, 3π/2]; su tale intervallo il seno è strettamente decrescente e la sua inversa
arcsen : [−1, 1] → [π/2, 3π/2] sarà una funzione strettamente decrescente e continua.
E6 Una discussione analoga si può fare per la funzione cos x con dominio [0, π] dove è strettamente decrescente. La sua immagine è [−1, 1] e la funzione inversa prende il nome di Arccos y
(“arcocoseno” o “ramo principale dell’arcocoseno”): il dominio è [−1, 1] e la sua immagine
19 Esercizio 12. Suggerimenti: 1) sen x > 0 per x ∈ (0, π/2] (segue da sen x = cos(x − π/2) e dalla
definizione di π/2); 2) cos x = 1 per x ∈ [0, π/2] implica x = 0 (cos x = 1 implica sen x = 0); 3) cos x è
iniettiva su [0, π/2] (se fosse cos x = cos y seguirebbe cos(x − y) = 1,..); 4) dalla Proposizione 37 segue che
cos x è strettamente decrescente in [0, π/2]); 5) sen x è strettamente crescente in [0, π/2]; etc.
16
Università “Roma Tre” – L. Chierchia
[0, π] e per la Proposizione 34 è una funzione continua. Gli altri “rami” dell’arcocoseno si ottengono considerando altri domini (massimali) su cui il coseno è invertibile: ad esempio, [π, 2π]; su
tale intervallo il coseno è strettamente crescente e la sua inversa arcocosen : [−1, 1] → [π, 2π]
sarà una funzione strettamente crescente e continua.
E7 La funzione tan x con dominio (−π/2, π/2) è strettamente crescente (e quindi iniettiva).
Poiché
sup tanh x = +∞ e
inf
tan x = −∞, per il teorema 35, la sua immagine è
(−π/2,π/2)
(−π/2,π/2)
R. La funzione inversa prende il nome di Arctan y (“arcotangente” o “ramo principale dell’arcotangente”): il suo dominio è R, è strettamente crescente e la sua immagine è (−π/2, π/2).
Per la Proposizione 34, Arctan y è continua. Gli altri “rami” dell’arcotangente si ottengono
considerando altri domini (massimali) su cui la tangente è invertibile: ad esempio, [π/2, 3π/2].
E8 Il teorema dei valori intermedi permette di definire in modo alternativo funzioni inverse
elementari, quali le radici o i logaritmi: ad esempio poiché la funzione x ∈ [0, +∞) �→ xn
(con n ∈ N) è strettamente crescente, per il Teorema dei valori intermedi, essa assume tutti
i valori tra 0 e +∞: questo significa che per ogni y ≥ 0 esiste (ed è unico) x ≥ 0 tale che
xn = y (e tale x è, per definizione, la radice n–sima di y e si denota con y 1/n ). Questa è una
dimostrazione diversa da quella data dell’esistenza delle radice ennesime. Dalla continuità di
xn e dalla Proposizione 38 segue poi che y 1/n è continua su [0, +∞).
5.5
Uniforme continuità
Definizione 40 Una funzione f : E → R si dice uniformemente continua su E se
∀ε>0,
∃δ>0
t.c.
|f (x) − f (y)| < ε ,
∀ x, y ∈ E , |x − y| < δ .
(19)
Si noti la differenza con la definizione di continuità in (15): per funzioni solo continue il δ
dipende sia da ε che da y (ossia δ = δ(ε, y)), mentre per funzione uniformemente continue il
δ dipende solo da ε ed è uniforme in y (ossia non vi dipende): δ = δ(ε).
Ad esempio (Esercizio 13) si ha che: f (x) = 1/x è continua su A = (0, 1) ma non uniformemente continua; la funzione f (x) = x è uniformemente continua su R; la funzione f (x) = x2
non è uniformemente continua su R.
Teorema 41 Sia f ∈ C(E) con E compatto. Allora, f è uniformemente continua su E.
Dimostrazione Supponiamo, per assurdo, che f non sia uniformemente continua su E.
Allora, esisterebbe ε > 0 tale che, per ogni δ > 0 esisterebbero x e y ∈ E con |x − y| < δ
ma |f (x) − f (y)| ≥ ε. Quindi, scegliendo δ = 1/n, si potrebbero trovare xn e yn ∈ E con
|xn − yn | < 1/n e |f (xn ) − f (yn )| ≥ ε. Poichè E è compatto, esisterebbe una sottosuccesione
{xnk } convergente ad un x̄ ∈ E. Ma, siccome |xn − yn | → 0, ne seguirebbe che
|ynk − x̄| ≤ |ynk − xnk | + |xnk − x̄| → 0 ,
e quindi anche ynk → x̄. Allora, avremmo la seguente contraddizione:
0 = |f (x̄) − f (x̄)| = lim |f (xnk ) − f (ynk )| ≥ ε > 0 .
k→∞
Scarica